www.vorhilfe.de
Vorhilfe

Kostenlose Kommunikationsplattform für gegenseitige Hilfestellungen.
Hallo Gast!einloggen | registrieren ]
Startseite · Forum · Wissen · Kurse · Mitglieder · Team · Impressum
Forenbaum
^ Forenbaum
Status Vorhilfe
  Status Geisteswiss.
    Status Erdkunde
    Status Geschichte
    Status Jura
    Status Musik/Kunst
    Status Pädagogik
    Status Philosophie
    Status Politik/Wirtschaft
    Status Psychologie
    Status Religion
    Status Sozialwissenschaften
  Status Informatik
    Status Schule
    Status Hochschule
    Status Info-Training
    Status Wettbewerbe
    Status Praxis
    Status Internes IR
  Status Ingenieurwiss.
    Status Bauingenieurwesen
    Status Elektrotechnik
    Status Maschinenbau
    Status Materialwissenschaft
    Status Regelungstechnik
    Status Signaltheorie
    Status Sonstiges
    Status Technik
  Status Mathe
    Status Schulmathe
    Status Hochschulmathe
    Status Mathe-Vorkurse
    Status Mathe-Software
  Status Naturwiss.
    Status Astronomie
    Status Biologie
    Status Chemie
    Status Geowissenschaften
    Status Medizin
    Status Physik
    Status Sport
  Status Sonstiges / Diverses
  Status Sprachen
    Status Deutsch
    Status Englisch
    Status Französisch
    Status Griechisch
    Status Latein
    Status Russisch
    Status Spanisch
    Status Vorkurse
    Status Sonstiges (Sprachen)
  Status Neuerdings
  Status Internes VH
    Status Café VH
    Status Verbesserungen
    Status Benutzerbetreuung
    Status Plenum
    Status Datenbank-Forum
    Status Test-Forum
    Status Fragwürdige Inhalte
    Status VH e.V.

Gezeigt werden alle Foren bis zur Tiefe 2

Navigation
 Startseite...
 Neuerdings beta neu
 Forum...
 vorwissen...
 vorkurse...
 Werkzeuge...
 Nachhilfevermittlung beta...
 Online-Spiele beta
 Suchen
 Verein...
 Impressum
Das Projekt
Server und Internetanbindung werden durch Spenden finanziert.
Organisiert wird das Projekt von unserem Koordinatorenteam.
Hunderte Mitglieder helfen ehrenamtlich in unseren moderierten Foren.
Anbieter der Seite ist der gemeinnützige Verein "Vorhilfe.de e.V.".
Partnerseiten
Dt. Schulen im Ausland: Mathe-Seiten:

Open Source FunktionenplotterFunkyPlot: Kostenloser und quelloffener Funktionenplotter für Linux und andere Betriebssysteme
Forum "Uni-Stochastik" - 2 ZV gleichvert. => st. unabh?
2 ZV gleichvert. => st. unabh? < Stochastik < Hochschule < Mathe < Vorhilfe
Ansicht: [ geschachtelt ] | ^ Forum "Uni-Stochastik"  | ^^ Alle Foren  | ^ Forenbaum  | Materialien

2 ZV gleichvert. => st. unabh?: Frage (beantwortet)
Status: (Frage) beantwortet Status 
Datum: 20:05 So 26.06.2011
Autor: Manu87

$ [mm] \Omega [/mm] = [mm] \{1, 2, 3, 4\}$ [/mm]
P ist gleichverteilt, also [mm] $P(X)=P(Y)=\frac{1}{|\Omega|}$ [/mm]

Ziel meiner Aufgabe ist zu zeigen, dass Korellationskoeffizient [mm] \rho [/mm] = 0 ist. Da Kovarianz bei stochastisch unabhängigen ZV 0 ist, folgt ja auch [mm] \rho [/mm] = 0.

Liege ich richtig in der Annahme, dass wenn 2 ZV gleichverteilt sind, sie automatisch stochastisch unabhängig sind? P ist ja fix, irgendwie sehe ich gar keine Möglichkeit einer stochastischen Abhängigkeit.


Grüße

        
Bezug
2 ZV gleichvert. => st. unabh?: Antwort
Status: (Antwort) fertig Status 
Datum: 21:22 So 26.06.2011
Autor: Al-Chwarizmi


> [mm]\Omega = \{1, 2, 3, 4\}[/mm]
>  P ist gleichverteilt, also
> [mm]P(X)=P(Y)=\frac{1}{|\Omega|}[/mm]
>  
> Ziel meiner Aufgabe ist zu zeigen, dass
> Korellationskoeffizient [mm]\rho[/mm] = 0 ist. Da Kovarianz bei
> stochastisch unabhängigen ZV 0 ist, folgt ja auch [mm]\rho[/mm] =
> 0.
>  
> Liege ich richtig in der Annahme, dass wenn 2 ZV
> gleichverteilt sind, sie automatisch stochastisch
> unabhängig sind? P ist ja fix, irgendwie sehe ich gar
> keine Möglichkeit einer stochastischen Abhängigkeit.
>  
>
> Grüße


Hallo Manu87,

ich denke, dass dies keine klar gestellte Aufgabe ist.
Offenbar möchtest du von zwei Zufallsgrößen X und Y
sprechen, von denen jede die Werte aus [mm] \Omega [/mm] an-
nehmen kann. Ferner sollen X und Y gleichverteilt
sein, also [mm] P(X=1)=P(X=2)=P(X=3)=P(X=4)=\frac{1}{4} [/mm]
und [mm] P(Y=1)=P(Y=2)=P(Y=3)=P(Y=4)=\frac{1}{4} [/mm] .

Dies kann man aber nur vermuten,
denn du sagst es nicht wirklich. (!!)

Die Antwort auf die so klar gestellte Frage:
Unter diesen (und keinen weiteren zusätzlichen)
Voraussetzungen folgt keineswegs, dass X und Y
unabhängig oder unkorreliert sind.

LG   Al-Chw.  


Bezug
                
Bezug
2 ZV gleichvert. => st. unabh?: Frage (beantwortet)
Status: (Frage) beantwortet Status 
Datum: 00:30 Mo 27.06.2011
Autor: Manu87

Aufgabe
Sei [mm] $\Omega [/mm]  = [mm] \{1,2,3,4\}$ [/mm] und P die Gleichverteilung auf [mm] \Omega. [/mm] Die Zufallsvariablen X und Y seien gegeben durch
[mm] \omega [/mm]  1, 2, 3, 4
X  1, 1,-1,-1
Y  2,-2, 1,-1
Zeigen Sie, dass $X = [mm] \frac{2}{3}Y^2-\frac{5}{3}$ [/mm] ist und dass [mm] $\rho(X, [/mm] Y) = 0$ gilt.

So war die Aufgabe. Ich dachte mir wenn P für alle fix ist, kann da auch keine Abh. sein oder?

Btw:Ich glaube X oder Y nimmt nicht [mm] \omega [/mm] an. Ich denke hier soll es so gehen:$ X: [mm] \omega \mapsto \IZ$ [/mm]

Bezug
                        
Bezug
2 ZV gleichvert. => st. unabh?: Antwort
Status: (Antwort) fertig Status 
Datum: 01:28 Mo 27.06.2011
Autor: Schadowmaster


> Btw:Ich glaube X oder Y nimmt nicht [mm]\omega[/mm] an. Ich denke
> hier soll es so gehen:[mm] X: \omega \mapsto \IZ[/mm]

Stimmt. ;)
Es ist X(1) = 1, X(2) = 1, X(3) = -1, etc.

Das behaupte ich ganz dreist, weil man damit die erste der zu zeigenden Bedingungen zeigen kann.

Aber ich frage mich gerade was das P - die Gleichverteilung - da zu suchen hat, also wo in der Aufgabe die gebraucht wird...


Bezug
                        
Bezug
2 ZV gleichvert. => st. unabh?: unkorreliert ≠ unabhängig
Status: (Antwort) fertig Status 
Datum: 02:02 Mo 27.06.2011
Autor: Al-Chwarizmi


> Sei [mm]\Omega = \{1,2,3,4\}[/mm] und P die Gleichverteilung auf
> [mm]\Omega.[/mm] Die Zufallsvariablen X und Y seien gegeben durch
>  [mm]\omega[/mm]  1, 2, 3, 4
>  X  1, 1,-1,-1
>  Y  2,-2, 1,-1
>  Zeigen Sie, dass [mm]X = \frac{2}{3}Y^2-\frac{5}{3}[/mm] ist und
> dass [mm]\rho(X, Y) = 0[/mm] gilt.
>  So war die Aufgabe. Ich dachte mir wenn P für alle fix
> ist, kann da auch keine Abh. sein oder?
>  
> Btw:Ich glaube X oder Y nimmt nicht [mm]\omega[/mm] an. Ich denke
> hier soll es so gehen:[mm] X: \omega \mapsto \IZ[/mm]


Hallo,

ist dir wenigstens bewusst, dass die ursprüngliche "Aufgabe"
mit dem, was du hier berichtest, keinerlei Zusammenhang
hat ?
So wie es jetzt aussieht, sollen X und Y offenbar zwei Funktionen
der einzigen wirklichen Zufallsvariablen [mm] \omega [/mm] mit Werten in
[mm] \Omega=\{1,2,3,4\} [/mm]  sein.
Die Tatsache (die noch zu beweisen ist !), dass stets [mm] x=\frac{2}{3}\,y^2-\frac{5}{3} [/mm]
ist, zeigt, dass x offenbar vom jeweiligen Wert von y abhän-
gig ist.
Der "Witz" der Aufgabe soll nun offenbar die Einsicht sein,
dass trotz dieser Abhängigkeit die "Korrelation" der Zufalls-
variablen X und Y verschwinden soll. Dies muss man einfach
nachrechnen und so bestätigen.
Die Lehre aus dem Beispiel soll wohl sein, dass aus der
Gleichung  "Korrelation(X,Y)=0"  nicht die stochastische
Unabhängigkeit der beiden betrachteten Zufallsvariablen X und Y
folgt.

LG   Al-Chw.


Bezug
                                
Bezug
2 ZV gleichvert. => st. unabh?: Frage (beantwortet)
Status: (Frage) beantwortet Status 
Datum: 15:22 Mo 27.06.2011
Autor: Manu87

Oops dann gibts wohl keine Punkte für mich. Ich werd das heute abend noch einmal nachrechnen. Danke für eure Hilfe.

Aber kann man das überhaupt so sagen?

Geben seinen in einem Gedankenmodell 2 Würfel auf [mm] \Omega [/mm] = {1,...,6}.
Ein roter stinknormaler Würfel mit Augen von 1 - 6 und ein blauer mit 2,4,6,8,10,12  Augen auf den Seiten.

[mm] $X_{rot} [/mm] : [mm] \Omega \mapsto \IZ$ [/mm]
[mm] $X_{rot} [/mm] : [mm] \omega \mapsto \omega$ [/mm]

[mm] $X_{blau} [/mm] : [mm] \Omega \mapsto \IZ$ [/mm]
[mm] $X_{blau} [/mm] : [mm] \omega \mapsto 2\cdot\omega$ [/mm]

So jetz haben wir 2 Würfel die scheinbar was gemeinsam haben. Und zwar
[mm] $$X_{rot}(\omega)=\frac{X_{blau}(\omega)}{2}$$ [/mm] und [mm] $$X_{blau}(\omega)=2\cdot X_{rot}(\omega)$$ [/mm]

Trotzdem sehe ich keine Abhängigkeit, da den einen Würfel kaum interessert was der andere macht...

[mm] $$P(X_{blau} \cap X_{rot}) [/mm] = [mm] P(X_{blau}) \cdot P(X_{rot})$$ [/mm]

Gilt ja. Das stelle ich mir so vor: für beide X soll [mm] $\omega [/mm] = 1$ gelten. Also rot zeigt 1 und blau zeigt 2. Folglich gilt für beide Seiten [mm] $\frac{1}{36}$. [/mm] Und keiner beienflusst den anderen.

Istdas so richtig? Wenn nein wo mache ich den Gedankenfehler?

Bezug
                                        
Bezug
2 ZV gleichvert. => st. unabh?: Antwort
Status: (Antwort) fertig Status 
Datum: 16:08 Mo 27.06.2011
Autor: Al-Chwarizmi


> Geben seinen in einem Gedankenmodell 2 Würfel auf [mm]\Omega[/mm] =
> {1,...,6}.
>  Ein roter stinknormaler Würfel mit Augen von 1 - 6 und
> ein blauer mit 2,4,6,8,10,12  Augen auf den Seiten.
>  
> [mm]X_{rot} : \Omega \mapsto \IZ[/mm]
>  [mm]X_{rot} : \omega \mapsto \omega[/mm]
>  
> [mm]X_{blau} : \Omega \mapsto \IZ[/mm]
>  [mm]X_{blau} : \omega \mapsto 2\cdot\omega[/mm]
>  
> So jetz haben wir 2 Würfel die scheinbar was gemeinsam
> haben. Und zwar
>  [mm]X_{rot}(\omega)=\frac{X_{blau}(\omega)}{2}[/mm] und
> [mm]X_{blau}(\omega)=2\cdot X_{rot}(\omega)[/mm]
>  
> Trotzdem sehe ich keine Abhängigkeit, da den einen Würfel
> kaum interessert was der andere macht...
>  
> [mm]P(X_{blau} \cap X_{rot}) = P(X_{blau}) \cdot P(X_{rot})[/mm]
>  
> Gilt ja. Das stelle ich mir so vor: für beide X soll
> [mm]\omega = 1[/mm] gelten. Also rot zeigt 1 und blau zeigt 2.    [haee]

du hast gerade gesagt: den einen Würfel interessiert kaum
was der andere macht - wenn rot 1 zeigt, dann könnte doch nun
blau ebensogut auch 6 oder 10 zeigen ...

> Folglich gilt für beide Seiten [mm]\frac{1}{36}[/mm]. Und keiner
> beienflusst den anderen.
>
> Istdas so richtig? Wenn nein wo mache ich den
> Gedankenfehler?


Mein(e) liebe(r) Manu,

das wäre jetzt halt wieder was anderes. In der ersten
Aufgabe (hier formuliert) wurde nur ein Würfel geworfen
und die Zufallsvariablen X und Y in Abhängigkeit des
Wurfergebnisses definiert.
Wenn du nun zwei Würfel nimmst, ist das einfach
eine neue Fragestellung, die mit der alten eigentlich
nichts zu tun hat.

LG   Al-Chw.


Bezug
Ansicht: [ geschachtelt ] | ^ Forum "Uni-Stochastik"  | ^^ Alle Foren  | ^ Forenbaum  | Materialien


^ Seitenanfang ^
www.vorhilfe.de